AM-GM Problem Limbo

Algebra Level 3

For positive real numbers a , a, b , b, and c , c, what is the minimum value of a b + c + b c + a + c a + b ? \frac{a}{b+c}+\frac{b}{c+a}+\frac{c}{a+b}?


The answer is 1.5.

This section requires Javascript.
You are seeing this because something didn't load right. We suggest you, (a) try refreshing the page, (b) enabling javascript if it is disabled on your browser and, finally, (c) loading the non-javascript version of this page . We're sorry about the hassle.

8 solutions

Shubhendra Singh
Dec 3, 2014

Here's one of the way other than using AM-GM .

Let a b c , a \geq b \geq c, so 1 b + c 1 a + c 1 a + b . \dfrac{1}{b+c}\geq\dfrac{1}{a+c}\geq \dfrac{1}{a+b}.

Now using the Rearrangement Inequality , we have

a b + c + b a + c + c b + a b b + c + c a + c + a b + a . . . . . . . . . . ( 1 ) \dfrac{a}{b+c}+\dfrac{b}{a+c}+\dfrac{c}{b+a}\geq\dfrac{b}{b+c}+\dfrac{c}{a+c}+\dfrac{a}{b+a}..........(1)

a b + c + b a + c + c b + a c b + c + a a + c + b b + a . . . . . . . . . . ( 2 ) \dfrac{a}{b+c}+\dfrac{b}{a+c}+\dfrac{c}{b+a}\geq\dfrac{c}{b+c}+\dfrac{a}{a+c}+\dfrac{b}{b+a}..........(2)

Add ( 1 ) (1) and ( 2 ) (2) to get

2 ( a b + c + b a + c + c b + a ) b + c b + c + a + c a + c + a + b b + a 2(\dfrac{a}{b+c}+\dfrac{b}{a+c}+\dfrac{c}{b+a})\geq\dfrac{b+c}{b+c}+\dfrac{a+c}{a+c}+\dfrac{a+b}{b+a}

2 ( a b + c + b a + c + c b + a ) 3 2(\dfrac{a}{b+c}+\dfrac{b}{a+c}+\dfrac{c}{b+a})\geq3

( a b + c + b a + c + c b + a ) 3 2 (\dfrac{a}{b+c}+\dfrac{b}{a+c}+\dfrac{c}{b+a})\geq \boxed{\dfrac{3}{2}}

with equality occurring where a = b = c . a=b=c.

Nesbitts Inequality !

This can be proved in a number of ways...including AM-GM, Titu's Lemma , and Jensen's Inequality .

Anyway, excellent use of rearrangement inequality!! :D

Aritra Jana - 6 years, 6 months ago

Let f ( a , b , c ) = c y c a b + c f(a,b,c)=\sum_{cyc}\frac{a}{b+c}

Note that f ( a , b , c ) f(a,b,c) is homogeneous of degree 0

So,we may assume a + b + c = 1 a+b+c=1

Hence, f ( a , b , c ) = c y c a b + c = c y c ( a + b + c b + c 1 ) f(a,b,c)=\sum_{cyc}\frac{a}{b+c}=\sum_{cyc}(\frac{a+b+c}{b+c}-1)

= c y c 1 a + b 3 9 2 3 =\sum_{cyc}\frac{1}{a+b}-3≥\frac{9}{2}-3 (By AM-HM)

Souryajit Roy - 6 years, 6 months ago

In many questions sometimes i add 2 inequalities , the answer gets wrong , so when two inequalities can be added?

U Z - 6 years, 6 months ago

Log in to reply

I think we can do so when the equality cases are the same...

Satvik Golechha - 6 years, 6 months ago
Joel Tan
Dec 6, 2014

By Cauchy-Schwarz ,

( ( a + b ) + ( b + c ) + ( c + a ) ) ( 1 a + b + 1 b + c + 1 c + a ) 9 ((a+b)+(b+c)+(c+a))(\frac {1}{a+b}+\frac {1}{b+c}+\frac {1}{c+a}) \geq 9 . Dividing by 2 and subtracting 3 gives the minimum of 1.5 when a = b = c a=b=c .

Vu Vincent
Aug 29, 2017

Let S = a b + c + b c + a + c a + b S=\frac{a}{b+c}+\frac{b}{c+a}+\frac{c}{a+b}

We know that (by AM-GM):

a + b b + c + b + c c + a + c + a a + b 3 \frac{a+b}{b+c}+\frac{b+c}{c+a}+\frac{c+a}{a+b} \ge 3 ( a b + c ) + b b + c + ( b c + a ) + c c + a + ( c a + b ) + a a + b 3 (\frac{a}{b+c})+\frac{b}{b+c}+(\frac{b}{c+a})+\frac{c}{c+a}+(\frac{c}{a+b})+\frac{a}{a+b} \ge 3 S + b b + c + c c + a + a a + b 3 \Rightarrow S+\frac{b}{b+c}+\frac{c}{c+a}+\frac{a}{a+b} \ge 3 S + ( 1 c b + c ) + ( 1 a c + a ) + ( 1 b a + b ) 3 \Rightarrow S+(1-\frac{c}{b+c})+(1-\frac{a}{c+a})+(1-\frac{b}{a+b}) \ge 3 S ( c b + c + a c + a + b a + b ) 0 \Rightarrow S-(\frac{c}{b+c}+\frac{a}{c+a}+\frac{b}{a+b}) \ge 0 S c b + c + a c + a + b a + b \Rightarrow S \ge \frac{c}{b+c}+\frac{a}{c+a}+\frac{b}{a+b} a b + c + b c + a + c a + b c b + c + a c + a + b a + b I \Leftrightarrow \frac{a}{b+c}+\frac{b}{c+a}+\frac{c}{a+b} \ge \frac{c}{b+c}+\frac{a}{c+a}+\frac{b}{a+b} \boxed{I}

The minimum occurs when L H S = R H S LHS = RHS . We see that the equality case of I \boxed{I} is a cyclic equation, from which we deduce that a = b = c a=b=c

Therefore, the minimum value is:

m i n ( S ) = 3 2 \boxed{min(S) = \frac{3}{2} }

Wonderful solution!

Nwankpa Richard - 3 years, 9 months ago

How were you able to deduce a = b = c a=b=c

Krish Shah - 1 year, 1 month ago
Andro Dellosa
Dec 29, 2014

Nesbitt's inequality... where k=3/2.

Son Nguyen
Jan 15, 2016

a b + c + b c + a + c a + b + 3 9 2 \frac{a}{b+c}+\frac{b}{c+a}+\frac{c}{a+b}+3\geq \frac{9}{2} 2 ( a + b + c ) ( 1 a + b + 1 b + c + 1 c + a ) 9 \Leftrightarrow 2(a+b+c)(\frac{1}{a+b}+\frac{1}{b+c}+\frac{1}{c+a})\geq 9 ( a + b + b + c + c + a ) ( 1 a + b + 1 b + c + 1 c + a ) 9 \Leftrightarrow (a+b+b+c+c+a)(\frac{1}{a+b}+\frac{1}{b+c}+\frac{1}{c+a})\geq 9 (Always happen)

Potato Magician
May 17, 2020

by AM-GM,

a b + c \frac{a}{b+c} + b c + a \frac{b}{c+a} + c a + b \frac{c}{a+b} \geq 3 × \times a b c ( a + b ) ( b + c ) ( c + a ) 3 \sqrt[3]{\frac{abc}{(a+b)(b+c)(c+a)}}

The expression is at minimum when a = b = c, therefore 3 × \times a b c ( a + b ) ( b + c ) ( c + a ) 3 \sqrt[3]{\frac{abc}{(a+b)(b+c)(c+a)}} = 3 × 3\times a 3 ( 2 a ) ( 2 a ) ( 2 a ) 3 \sqrt[3]{\frac{a^3}{(2a)(2a)(2a)}} = 3 × 1 2 3\times\frac{1}{2} = 1.5

Krish Shah
Apr 15, 2020

Starting with a b + c + b c + a + c a + b \frac{a}{b+c} + \frac{b}{c+a} + \frac{c}{a+b} , if we add and subtract one from each term, we get

a + b + c b + c + a + b + c c + a + a + b + c a + b 3 \frac{a+b+c}{b+c} + \frac{a+b+c}{c+a} + \frac{a+b+c}{a+b} - 3

= ( a + b + c ) ( 1 b + c + 1 c + a + 1 a + b ) 3 =(a+b+c)(\frac{1}{b+c} + \frac{1}{c+a} + \frac{1}{a+b}) - 3

= 0.5 ( ( b + c ) + ( c + a ) + ( a + b ) ) ( 1 b + c + 1 c + a + 1 a + b ) 3 =0.5((b+c)+(c+a)+(a+b))(\frac{1}{b+c} + \frac{1}{c+a} + \frac{1}{a+b}) - 3

now applying Cauchy-Schwarz , we get

0.5 ( 3 2 ) 3 ≥0.5(3^2) - 3

3 2 ≥\frac{3}{2}

Thus, the minimum of the expression is 3 2 \frac{3}{2} .

Sita Ram
Feb 25, 2017

Let minimum be 1 put get 1.5

0 pending reports

×

Problem Loading...

Note Loading...

Set Loading...